Difference between revisions of "2021 AMC 12A Problems/Problem 25"
Lopkiloinm (talk | contribs) (→Solution) |
Lopkiloinm (talk | contribs) (→Solution) |
||
Line 7: | Line 7: | ||
Suppose a counting number x be not divisible by 3. Multiply x by 9. Multiplying x by 9 adds a set <math>d(x)</math> divisors that are the original divisors multiply by 3 and an additional <math>d(x)</math> divisors that are the originals multiplied by 9 which end up saying that <math>d(9x) = 3d(x)</math>. Another consequence is multiplying the denominator by <math>{\sqrt[3]{9}}</math>. So now <math>f(9x) > f(x)</math> because <math>\frac{d(9x)}{\sqrt[3]{9x}}= \frac{d(x)}{\sqrt[3]{x}}\frac{3}{\sqrt[3]{9}} > \frac{d(x)}{\sqrt[3]{x}}</math> because <math>3 > \sqrt[3]{9}</math>. A property of multiples of 9 is their digits add up to multiples of 9, so the only possibility is <math>\boxed{(E) 9}</math> | Suppose a counting number x be not divisible by 3. Multiply x by 9. Multiplying x by 9 adds a set <math>d(x)</math> divisors that are the original divisors multiply by 3 and an additional <math>d(x)</math> divisors that are the originals multiplied by 9 which end up saying that <math>d(9x) = 3d(x)</math>. Another consequence is multiplying the denominator by <math>{\sqrt[3]{9}}</math>. So now <math>f(9x) > f(x)</math> because <math>\frac{d(9x)}{\sqrt[3]{9x}}= \frac{d(x)}{\sqrt[3]{x}}\frac{3}{\sqrt[3]{9}} > \frac{d(x)}{\sqrt[3]{x}}</math> because <math>3 > \sqrt[3]{9}</math>. A property of multiples of 9 is their digits add up to multiples of 9, so the only possibility is <math>\boxed{(E) 9}</math> | ||
− | Edit: It seems that this proof is not complete because we also need to check whether multiply by 3 is better than multiply by 9. It is better to multiply by 9 than by 3 shown by similar logic | + | Edit: It seems that this proof is not complete because we also need to check whether multiply by 3 is better than multiply by 9. It is better to multiply by 9 than by 3 shown by similar logic which I will leave as an exercise for the reader. |
~Lopkiloinm | ~Lopkiloinm | ||
Revision as of 17:56, 11 February 2021
Problem
Let denote the number of positive integers that divide , including and . For example, and . (This function is known as the divisor function.) LetThere is a unique positive integer such that for all positive integers . What is the sum of the digits of
Solution
Suppose a counting number x be not divisible by 3. Multiply x by 9. Multiplying x by 9 adds a set divisors that are the original divisors multiply by 3 and an additional divisors that are the originals multiplied by 9 which end up saying that . Another consequence is multiplying the denominator by . So now because because . A property of multiples of 9 is their digits add up to multiples of 9, so the only possibility is
Edit: It seems that this proof is not complete because we also need to check whether multiply by 3 is better than multiply by 9. It is better to multiply by 9 than by 3 shown by similar logic which I will leave as an exercise for the reader. ~Lopkiloinm
See also
2021 AMC 12A (Problems • Answer Key • Resources) | |
Preceded by Problem 24 |
Followed by Last problem |
1 • 2 • 3 • 4 • 5 • 6 • 7 • 8 • 9 • 10 • 11 • 12 • 13 • 14 • 15 • 16 • 17 • 18 • 19 • 20 • 21 • 22 • 23 • 24 • 25 | |
All AMC 12 Problems and Solutions |
The problems on this page are copyrighted by the Mathematical Association of America's American Mathematics Competitions.